LSAT and Law School Admissions Forum

Get expert LSAT preparation and law school admissions advice from PowerScore Test Preparation.

User avatar
 Dave Killoran
PowerScore Staff
  • PowerScore Staff
  • Posts: 5853
  • Joined: Mar 25, 2011
|
#87762
Complete Question Explanation
(The complete setup for this game can be found here: lsat/viewtopic.php?f=155&t=1451)

The correct answer choice is (D)

This is a tricky question. The question stem focuses on O, which is connected to R from the first rule. Thus, one would expect that the later O auditions, the more problems that could occur. Thus, the first three answers—which each include Wednesday—seem the least suspicious. Between the last two answers, (D) seems more problematic. Why? Because answer choice (E)—Thursday and Saturday—would fulfill the third rule, reducing the number of concerns in placing the actors. Thus, answer choice (D) appears to be the most likely correct answer based upon an initial analysis of the answers.

In answer choice (D), if O auditions on Thursday and Friday, then from the first rule R must audition on Friday and Saturday. That leaves G to fulfill the third rule and audition on Thursday and Saturday. However, this violates the condition that an actor must audition each day, because there is no actor available to audition on Wednesday. Hence, answer choice (D) cannot occur and is correct.
 LSAT2018
  • Posts: 242
  • Joined: Jan 10, 2018
|
#46738
Would this question be answered directly from an inference made during the setup? I was able to eliminate some based on Questions 6 and 7, eliminating (B) and (E) respectively, but was left with three remaining.
 Jon Denning
PowerScore Staff
  • PowerScore Staff
  • Posts: 904
  • Joined: Apr 11, 2011
|
#46976
Hi LSAT - probably not, honestly. This would be an awfully tricky inference to catch in the setup, and not even the sort of thing most people (myself included) would really be looking for: the idea of two days that O cannot audition is a question-driven consideration, not a global one unless you just happened to get lucky and catch it. And even then it's not the kind of thing you'd diagram, but rather just an interesting truth to keep in mind in case it comes up down the line.

The reason D is correct here btw is that with O on Thursday and Friday you'd also then have to put R on Friday and Saturday to keep the first O ahead of the first R. That means the variables to go on both Thursday and Saturday (the last rule) must be G...but that leaves the first day, Wednesday, wide open and thus violates the scenario we're given.

But again hardly the kind of insight that could be reasonably expected up front. More the kind of thing you'd simply have to investigate and discover once it's asked of you.

Get the most out of your LSAT Prep Plus subscription.

Analyze and track your performance with our Testing and Analytics Package.